52
$\begingroup$

Let $a,b$ be positive integers. Because binomial coefficients are integers, we know that $a!b!$ divides $(a+b)!$. For particular $a$ and $b$ there may be a gap $g$ with a tighter result, so $a!b!$ divides $(a+b-g)!$. To give two examples:

  • $6!7!$ divides $10!$ (so the gap is 3)
  • $187!239!$ divides $416!$ (so the gap is 10)

Let us define the gap $g$ for $a$ and $b$ as $$ g(a,b) = \max\{\,w\in\Bbb N : a!\,b!\mid (a+b-w)!\}$$ We can plot the gap $g(a,b)$ as $a$ and $b$ vary. The plot is symmetric in $a$ and $b$, so if we only plot one half, we produce a figure like this: Heatmap of the gap g(a,b)

What I would really like to ask is "What is going on with this figure?".

In the interests of providing an answerable question, let me instead ask "How often is the gap zero?" Specifically, can we estimate $$ z = \lim_{N\rightarrow\infty} \frac{\sum_{a=1}^{N}\sum_{b=1}^{N} \Large 𝟙_{g(a,b)=0}}{N^2}$$ In the figure above with $N=256$, 7.7% of the entries have $g(a,b)=0$.

Addendum: Mathworker21 made an interesting comment below that seems worth capturing. For any $a$, $$g(a,a!-1)=a-1$$ In particular, $g(a,b)$ grows unboundedly, and this class of examples in some sense achieves the fastest rate of growth.

Addendum #2: Tim Chow suggests considering one prime and asks what the $p$-adic version of the table looks like. Let $v_p()$ be the $p$-adic valuation and define $$g_p(a,b) = \max\{\,w\in\Bbb N : v_p(a!\,b!) \leq v_p((a+b-w)!)\}$$ This $g_p(a,b)$ is the same quantity considered in Kummer's theorem, which shows that $g_p(a,b)$ equals the number of carries when adding $a$ to $b$ in base-$p$ arithmetic.

Here are the corresponding (substantially cleaner) figures for $p=2,3,5$: enter image description here enter image description here enter image description here

$\endgroup$
17
  • 9
    $\begingroup$ No idea, I'm just here to say that the (rotated) serpinski triangles above the anti-diagonal look very cool. $\endgroup$ Commented May 17 at 21:03
  • 3
    $\begingroup$ I just wanted to mention that if $a! b! \mid n!$ then $a+b \le n + c\frac{\log n}{\log\log n}$ (since $a!b! \le n!$), and this bound is sometimes achieved (e.g. $b = a!-1$, $n = a!$). $\endgroup$ Commented May 17 at 23:21
  • 2
    $\begingroup$ @MichaelEngelhardt As my comment says, take $b = a! - 1$ and then $v = a-1$: we have $a! b! = (a!)!$ and $(a+b-v)! = (a!)!$. $\endgroup$ Commented May 18 at 9:24
  • 9
    $\begingroup$ @MaxLonysaMuller You asked where this question came from. Apparently in Chebyshev's preliminary work towards the Prime Number Theorem he used the fact that $[(30N)! N!] / [(15N)! (10N)! (6N!)]$ is an integer. If the numerator were $31N$, integrality would be trivial to prove, but his version is a little more finicky. The table in this post was an attempt to get a little more intuition on the situation. More details here: mathoverflow.net/questions/26336/… $\endgroup$ Commented May 18 at 14:39
  • 4
    $\begingroup$ @BillBradley The standard way to prove integrality of ratios of factorials is to consider one prime at a time. Have you tried plotting $p$-adic versions of your graph? $\endgroup$ Commented May 19 at 13:19

2 Answers 2

25
$\begingroup$

We indeed have $z = 0$. I don't think this at all answers the question "What is going on with this figure?".

Take $\varepsilon > 0$. We show the limsup is at most $\varepsilon$. Take large $C = C(\varepsilon) \in \mathbb{N}$. By a "union-bound", the number of pairs $a,b \le x$ with $\gcd(a,b) > C$ is at most $\sum_{g > C} \lfloor \frac{x}{C} \rfloor^2 \le \frac{10x^2}{C} \le \varepsilon x^2$.

So let's restrict to pairs $(a,b)$ with $\gcd(a,b) \le C$. This means there are only finitely many possible prime powers that divide both $a$ and $b$. For example, if $C = 10$, then the possibilities are $2^1, 2^2, 2^3, 3^1, 3^2, 5^1, 7^1$. Let $K \in \mathbb{N}$ be the largest exponent among these possible prime powers. Note that there are at most $C$ possible primes composing these prime powers, and note that $K \approx \log_2 C$.

Now, $a!b! \mid (a+b-1)!$ iff for each prime $p$, we have $$\sum_{k=1}^\infty \left(\left\lfloor \frac{a+b-1}{p^k} \right\rfloor - \left\lfloor \frac{a}{p^k} \right\rfloor - \left\lfloor \frac{b}{p^k} \right\rfloor\right) \ge 0.$$ The key (and trivial) observation is that \begin{equation}\tag{1}\label{1} \left\lfloor \frac{a+b-1}{p^k} \right\rfloor - \left\lfloor \frac{a}{p^k} \right\rfloor - \left\lfloor \frac{b}{p^k} \right\rfloor = \begin{cases} -1 & \text{if } r_a, r_b = 0 \\ 0 & \text{if } 0 < r_a + r_b < p^k \\ 1 & \text{if } r_a+r_b \ge p^k \end{cases}, \end{equation} where $r_a, r_b$ are the remainders when $a$ and $b$ are divided by $p^k$, respectively.

Let's focus on equation \eqref{1} for $k \le 100K^2$, say. For $k > 100K^2$, we don't get $-1$ (by the definition of $K$).

Since $100K^2$ is some fixed number, it holds as $x \to \infty$ that if we choose $(a,b) \in [x] \times [x]$ uniformly at random subject to a $\gcd$ constraint, then the residues $(r_a,r_b)$ are equidistributed (up to error $o_{x \to \infty}(1)$) in a suitable sense. A bit more precisely, once we condition on $\gcd(a,b) = g$ with $p^{k_0} \parallel g$, the residues $(r_a,r_b)$ modulo $p^{k_0+1}$ are equidistributed over the set $\{0, p^{k_0}, 2p^{k_0},\dots, (p-1)p^{k_0}\}$, and once we condition on the residues modulo $p^{k_0+1}$, the residues modulo $p^{k_0+2}$ are distributed uniformly over an analogous set. Etc. up to $p^{100K^2}$. Therefore, whether the sum of the residues is at least the modulus (i) has probability at least $\frac{1}{10}$, say and (ii) is independent of the previous residues.

[Remark: about (i), the probability is close to $1/2$ as $p$ increases, but can be smaller, for example if $p^{k_0} = 2$, then the residues modulo $p^{k_0+1} = 4$ are $\{0,2\}$, giving a probability of $\frac{1}{4}$].

Consequently, although in the worst case we get a sum of $-K$ for $k \in \{1,\dots,K\}$, we, with probability $1 - \exp(-K^2)$ (over the choice of $(a,b)$), end up with a non-negative (in fact, positive and large) sum for $k \in \{1,\dots,K^2\}$.

By union-bounding over all possible primes, of which there are at most $2^K$, we get probability at least $1 - o_K(1) \ge 1 - \varepsilon$, for $a!b! | (a+b-1)!$, as desired.

$\endgroup$
12
$\begingroup$

Erdős (Aufgabe 557, Elemente der Mathematik 23 (1968), 111-113) proved that if $a!b! \mid n!$ then

$$ a+b < n+O(\log n)$$

for all $n$ (this follows easily from considering $2$-adic valuations, and has been rediscovered many times, but I think this was the first place it appeared in the literature). Erdős also proved that this bound is sharp, in that there are infinitely many $n$ together with $a,b\leq n$ such that $a!b!\mid n!$ and $a+b\geq n+c\log n$ for some absolute constant $c>0$.

(This contradicts the bound of $a+b< n+O(\frac{\log n}{\log\log n})$ claimed by @mathworker21 in their comment, but I think they made a calculation error somewhere?)

This construction follows from the following stronger fact: for almost all $n$, we have that $n!(n+k)! \mid (2n)!$, where $k=\lfloor C\log n\rfloor$ for some suitable fixed constant $C>0$.

In your notation, this implies that for all pairs $a,b$ we have $$g(a,b)\ll \log(a+b)$$ and there are infinitely many pairs $a,b$ such that $a\asymp b$ and $g(a,b)\gg \log(a+b)$.

(I'll take this opportunity to also mention the lovely conjecture of Erdős, Graham, Ruzsa, and Straus that, for all $k\geq 2$, there are infinitely many $n$ such that $(n+k)!^2 \mid (2n)!$. Balakran (On the values of $n$ which make $(2n)!/(n+1)!(n+1)!$ an integer. J. Indian Math. Soc. (1929), 97-100) proved this is true for $k=1$. As far as I know it is open even for $k=2$. See Problem 727 on Erdos Problems.)

$\endgroup$
3
  • 6
    $\begingroup$ Related: mathoverflow.net/q/137920 $\endgroup$ Commented May 21 at 21:32
  • 1
    $\begingroup$ Does one even expect that $(n+k)!^2$ divides $(2n)!$ for a positive proportion of $n$ (for each fixed $k$)? Since for a positive proportion we should have all prime factors $p$ of $n+1,\dots,n+k$ at most $n^\delta$ for any fixed $\delta$ and that ensures that the base $p$ expression of $n$ has many digits so it very likely has a lot of carries. $\endgroup$ Commented May 26 at 15:55
  • 1
    $\begingroup$ That sounds reasonable to me. $\endgroup$ Commented May 27 at 7:13

You must log in to answer this question.

Start asking to get answers

Find the answer to your question by asking.

Ask question

Explore related questions

See similar questions with these tags.